Most opera singers who add demanding roles to their repertoires at a young age lose their voices early. It has been s...

Lily on August 6, 2013

Opera

Would you please diagram the conditionals so I can compare mine to them and also please explain why B is wrong and E is correct? Thank you in advance!

Replies
Create a free account to read and take part in forum discussions.

Already have an account? log in

Naz on August 7, 2013

Answer choice (A) is incorrect because we know nothing about young opera singers without great vocal power.

Answer choice (B) is in incorrect because it is merely restating something in the passage that the author doesn't agree with. Remember, the author uses, "it has been said," right before he states this line: "it has been said that this is because their voices have not yet matured and hence lack the power for such roles." The author continues with a contradictory conjunction "but," conveying that she does not agree and offers a different reason. Therefore, we cannot say that the answer choice (B) is most strongly supported by the information given to us.

Answer choice (C) is irrelevant because the author never discusses what singers should or should not do.

Answer choice (D) is incorrect because we know nothing about mature opera singers.

Answer choice (E) is our answer because we know that "most young singers do not have technical training," and since technical training is necessary not to strain your vocal chords, then: "if you avoid straining your vocal chords, then you are technically trained."

YS - most - not TT

ASV ==> TT
not TT==> not ASV

Remember to arrange like so: YS - most - not TT ==> not ASV

Therefore: YS - most - not ASV

I hope that helps! Let me know if you have any other questions.

Lily on August 8, 2013

Thank you ... I didn't see "it has been said" for B and on E I did not see how "demanding roles" ended up in the answer.

DennisGerrard on July 13, 2017

any similar problem?

Mehran on July 21, 2017

@DennisGerrard yes, this is a Quantifier so check out the Quantifier questions.

Hope that helps! Please let us know if you have any other questions.